Quantcast
  • Register
PhysicsOverflow is a next-generation academic platform for physicists and astronomers, including a community peer review system and a postgraduate-level discussion forum analogous to MathOverflow.

Welcome to PhysicsOverflow! PhysicsOverflow is an open platform for community peer review and graduate-level Physics discussion.

Please help promote PhysicsOverflow ads elsewhere if you like it.

News

PO is now at the Physics Department of Bielefeld University!

New printer friendly PO pages!

Migration to Bielefeld University was successful!

Please vote for this year's PhysicsOverflow ads!

Please do help out in categorising submissions. Submit a paper to PhysicsOverflow!

... see more

Tools for paper authors

Submit paper
Claim Paper Authorship

Tools for SE users

Search User
Reclaim SE Account
Request Account Merger
Nativise imported posts
Claim post (deleted users)
Import SE post

Users whose questions have been imported from Physics Stack Exchange, Theoretical Physics Stack Exchange, or any other Stack Exchange site are kindly requested to reclaim their account and not to register as a new user.

Public \(\beta\) tools

Report a bug with a feature
Request a new functionality
404 page design
Send feedback

Attributions

(propose a free ad)

Site Statistics

205 submissions , 163 unreviewed
5,047 questions , 2,200 unanswered
5,345 answers , 22,709 comments
1,470 users with positive rep
816 active unimported users
More ...

  Examples of number theory showing up in physics

+ 11 like - 0 dislike
3798 views

My question is very simple: Are there any interesting examples of number theory showing up unexpectedly in physics?

This probably sounds like rather strange question, or rather like one of the trivial to ask but often unhelpful questions like "give some examples of topic A occurring in relation to topic B", so let me try to motivate it.

In quantum computing one well known question is to quantify the number of mutually unbiased (orthonormal) bases (MUBs) in a $d$-dimensional Hilbert space. A set of bases is said to be mutually unbiased if $|\langle a_i | b_j \rangle|^2 = d^{-1}$ for every pair of vectors from chosen from different bases within the set. As each basis is orthonormal we also have $\langle a_i | a_j \rangle =\delta_{ij}$ for vectors within the same basis. We know the answer when $d$ is prime (it's $d+1$) or when $d$ is an exact power of a prime (still $d+1$), but have been unable to determine the number for other composite $d$ (even the case of $d=6$ is open). Further, there is a reasonable amount of evidence that for $d=6$ there are significantly less than $7$ MUBs. If correct, this strikes me as very weird. It feels (to me at least) like number theoretic properties like primality have no business showing up in physics like this. Are there other examples of this kind of thing showing up in physics in a fundamental way?


This post imported from StackExchange Physics at 2014-03-24 09:19 (UCT), posted by SE-user Joe Fitzsimons

asked Dec 1, 2011 in Mathematics by Joe Fitzsimons (3,575 points) [ revision history ]
retagged Mar 25, 2014 by dimension10
Most voted comments show all comments
The MUB connection is really to finite fields (and latin squares as @FrédéricGrosshans mentions) and only through that to prime numbers. I guess we could say this is a connection to number theory, but really seems like a connection to abstract algebra, which is not nearly as surprising.

This post has been migrated from (A51.SE)
The MUB connection is really to finite fields (and latin squares as @FrédéricGrosshans mentions) and only through that to prime numbers. I guess we could say this is a connection to number theory, but really seems like a connection to abstract algebra, which is not nearly as surprising.

This post imported from StackExchange Physics at 2014-03-24 09:19 (UCT), posted by SE-user Artem Kaznatcheev
@Artem: It's true that you can arrive at above result via finite fields, but the structure of the partial results is governed by number theoretic properties. I don't really see the way of arriving at a given result as particularly fundamental, as there are often multiple paths to the result.

This post has been migrated from (A51.SE)
@Artem: It's true that you can arrive at above result via finite fields, but the structure of the partial results is governed by number theoretic properties. I don't really see the way of arriving at a given result as particularly fundamental, as there are often multiple paths to the result.

This post imported from StackExchange Physics at 2014-03-24 09:19 (UCT), posted by SE-user Joe Fitzsimons
Duplicate on Phys.SE: physics.stackexchange.com/q/414/2451

This post imported from StackExchange Physics at 2014-03-24 09:19 (UCT), posted by SE-user Qmechanic
Most recent comments show all comments
MUBs is a really fascinating subjects. They are also linked with [Latin squares](http://en.wikipedia.org/wiki/Graeco-Latin_square#Mutually_orthogonal_Latin_squares), as e.g; shown [here](http://pra.aps.org/abstract/PRA/v79/i1/e012109). I find this link with number theory more surprising than the role of prime numbers.

This post has been migrated from (A51.SE)
MUBs is a really fascinating subjects. They are also linked with Latin squares, as e.g; shown here. I find this link with number theory more surprising than the role of prime numbers.

This post imported from StackExchange Physics at 2014-03-24 09:19 (UCT), posted by SE-user Frédéric Grosshans

11 Answers

+ 8 like - 0 dislike

There are many attempts for a physical proof of the Riemann hypothesis. The major work in this direction was summarized in a recent review by: Schumayer and Hutchinson.

One of these attempts was proposed by: Berry and Keating. Their suggestion is within the framework of the Hilbert–Pólya conjecture, according to which, the Hilbert–Pólya Hamiltonian, whose spectrum is the imaginary part of the zeta zeros, can be obtained by quantizing a classical Hamiltonian of a chaotic system having periodic orbits with log prime periods. They argue that the classical Hamiltonian can be $xp$ (with appropriate yet unknown boundary conditions).

Another suggestion is due to Freeman Dyson in his Birds and Frogs lecture who suggests that the Riemann hypothesis might be proved through the classification of one dimensional quasicrystals.

This post imported from StackExchange Physics at 2014-03-24 09:19 (UCT), posted by SE-user David Bar Moshe
answered Dec 1, 2011 by David Bar Moshe (4,355 points) [ no revision ]
This seems more like engineering a physical system to embody certain number theoretic properties, rather than them occurring unexpectedly.

This post imported from StackExchange Physics at 2014-03-24 09:19 (UCT), posted by SE-user Joe Fitzsimons
I wouldn't count it as "engineering". It's more like using physical intuition in order to make a breakthrough in maths. We know the properties the Hilbert-Polya hamiltonian should have, so we whether it might be implemented in a physical system.

This post imported from StackExchange Physics at 2014-03-24 09:19 (UCT), posted by SE-user Javier Rodriguez Laguna
+ 7 like - 0 dislike

Here is a toy example; I don't know how interesting this will be to physicists. The eigenvalues of the Laplacian acting on, say, smooth functions $\mathbb{R}^k/(2\pi \mathbb{Z})^k \to \mathbb{C}$ are given by $$\{ m_1^2 + ... + m_k^2 : m_i \in \mathbb{Z} \}.$$

as a multiset (that is, with multiplicities). These are the energy eigenvalues of $n$ free non-interacting quantum particles on a circle. The multiplicity of a given eigenvalue is therefore the number of ways to write it as a sum of $k$ (integer) squares.

This is a classical number-theoretic problem. For example, it is a classical result that the number of ways to write a non-negative integer $n$ as the sum of two squares is $$r_2(n) = 4 \sum_{d | n} \chi_4(d)$$

where $\chi_4(d)$ is equal to $0$ if $d \equiv 0, 2 \bmod 4$, equal to $1$ if $d \equiv 1 \bmod 4$, and equal to $-1$ if $d \equiv 3 \bmod 4$. In general, the number of ways $r_k(n)$ to write a non-negative integer $n$ as the sum of $k$ squares has generating function $$\sum r_k(n) q^n = \left( \sum_{m \in \mathbb{Z}} q^{m^2} \right)^k = \theta(q)^k.$$

The function $\theta(q)$ is a theta function. Theta functions are closely related to modular forms, an important topic in number theory, and in fact the classical proof of the closed form $$r_4(n) = 8 \sum_{d | n} [4 \nmid d]$$

(where we have used the Iverson bracket above) proceeds by showing that $\theta(q)^4$ is a modular form; see Wikipedia.

This post imported from StackExchange Physics at 2014-03-24 09:19 (UCT), posted by SE-user Qiaochu Yuan
answered Dec 1, 2011 by Qiaochu Yuan (385 points) [ no revision ]
The $\theta$ page is crowded with greeks like the acropolis. Can you specify which $\theta$ you mean?

This post imported from StackExchange Physics at 2014-03-24 09:19 (UCT), posted by SE-user draks ...
@draks: it's defined immediately before I use the symbol.

This post imported from StackExchange Physics at 2014-03-24 09:19 (UCT), posted by SE-user Qiaochu Yuan
Ah, that was too close for me, thanks. (i) How are $k$, the number of squares and $n$, the number of particles on the circle related? And (ii) are there comparable formulas for $r_k(n)$ when one sums powers other than 2?

This post imported from StackExchange Physics at 2014-03-24 09:19 (UCT), posted by SE-user draks ...
@draks: $k$ is the number of particles. $n$ is (up to some normalization) an energy eigenvalue. The situation when summing powers other than $2$ is considerably more complicated because the close relationship to modular forms disappears and I don't know what's known about it.

This post imported from StackExchange Physics at 2014-03-24 09:19 (UCT), posted by SE-user Qiaochu Yuan
Thanks a lot. $ $

This post imported from StackExchange Physics at 2014-03-24 09:19 (UCT), posted by SE-user draks ...
+ 6 like - 0 dislike

I have an example of my own :). It appeared trying to calculate the dimension of a Hilbert space associated with rotationally invariant systems of n spins. The dimension was given in terms of the Moebius function. for details, check the appendix of Phys. Rev. E 76, 061127 (2007) or arXiv:quant-ph/0702164.

This post imported from StackExchange Physics at 2014-03-24 09:19 (UCT), posted by SE-user user667
answered Dec 2, 2011 by anonymous [ no revision ]
That's weird, and certainly interesting. I'll give the paper a look.

This post imported from StackExchange Physics at 2014-03-24 09:19 (UCT), posted by SE-user Joe Fitzsimons
+ 5 like - 0 dislike

I've encoutered Diophantine equations (a variant of Pell's equation) in an (unpublished) attempt to turn a molecular system into a classical logical gate. The goal was to (approximately) synchronize incommensurable oscillations, and successives solution to the Diophantine equation gave me better fidelities.

I don't know if it qualifies for number theory, or even for physics, but I was surprised to find this equation as a good tool for my physics problem.

If anyone is interested, I can probably unearth my old notes and write something more detailed on the problem and the solution I found. Just ask in the comment.

This post imported from StackExchange Physics at 2014-03-24 09:19 (UCT), posted by SE-user Frédéric Grosshans
answered Dec 1, 2011 by Frédéric Grosshans (250 points) [ no revision ]
This is a general property--- diophantine approximation is related to resonance in classical systems like in KAM.

This post imported from StackExchange Physics at 2014-03-24 09:19 (UCT), posted by SE-user Ron Maimon
+ 3 like - 0 dislike

For quantized cat maps, the inverse of Planck's constant is an integer N . There are various results for the special cases, where N is a power of a prime. So, the arithmetic properties of N play an important role here.

For references, see http://www.math.kth.se/~rikardo/cat2.pdf .

This post has been migrated from (A51.SE)
answered Dec 1, 2011 by jjcale (70 points) [ no revision ]
Could you elaborate on this a bit?

This post has been migrated from (A51.SE)
Added a reference.

This post has been migrated from (A51.SE)
+ 3 like - 0 dislike

There are many theorems in quantum information which only apply to qudits of prime dimension. In particular, this seems to happen with graph states. In that case many theorems rely on the fact that multiplication modulo a prime is an invertible operation.

The Chinese Remainder Theorem can be used to show that graph states made of qudits of square-free dimension are equivalent to collections of graph states of qudits of prime dimension (the primes being the prime factorization of the original dimension).

Related to number theory is algebra. Group theory in particular tends to play an important role in quantum computing (e.g. the hidden subgroup problem).

This post imported from StackExchange Physics at 2014-03-24 09:19 (UCT), posted by SE-user Dan Stahlke
answered Dec 2, 2011 by Dan Stahlke (70 points) [ no revision ]
Thanks for taking the time to compose an answer. I don't really consider quantum algorithms as fundamental physics in the sense of this question, particularly given that the hidden subgroup stuff is driven by a generalization of problems from number theory (factoring/discrete logs). The graph state observation seems more related to the fact that you are looking at factoring a Hilbert space, which directly relates to primality of the dimesnionality, etc.

This post imported from StackExchange Physics at 2014-03-24 09:19 (UCT), posted by SE-user Joe Fitzsimons
+ 2 like - 0 dislike

This answer is closely related to jjcale's answer. In this article, Gurevich and Hadani prove Rudnick's quantum ergodicity conjecture about the Berry-Hannay model. To do it they construct a number-theoretical description of the quantization of a torus phase space at rational values of hbar, involving l-adic sheaves on an algebraic variety of positive characteristic.

This post has been migrated from (A51.SE)
answered Dec 3, 2011 by Squark (1,725 points) [ no revision ]
This sounds more like physics showing up in number theory.

This post has been migrated from (A51.SE)
@MBN not quite. The result they prove belongs to the realm of quantum dynamical systems, not number theory. Number theory, or, more precisely, arithmetic geometry is a tool to solve the problem.

This post has been migrated from (A51.SE)
+ 2 like - 0 dislike

This answer is closely related to jjcale's answer. In this article, Gurevich and Hadani prove Rudnick's quantum ergodicity conjecture about the Berry-Hannay model. To do it they construct a number-theoretical description of the quantization of a torus phase space at rational values of hbar, involving l-adic sheaves on an algebraic variety of positive characteristic.

This post imported from StackExchange Physics at 2014-03-24 09:19 (UCT), posted by SE-user Squark
answered Dec 3, 2011 by anonymous [ no revision ]
This sounds more like physics showing up in number theory.

This post imported from StackExchange Physics at 2014-03-24 09:19 (UCT), posted by SE-user MBN
@MBN not quite. The result they prove belongs to the realm of quantum dynamical systems, not number theory. Number theory, or, more precisely, arithmetic geometry is a tool to solve the problem.

This post imported from StackExchange Physics at 2014-03-24 09:19 (UCT), posted by SE-user Squark
+ 2 like - 0 dislike

in general the Riemann xi function can be proved to be a functonal determinant

$ \frac{\xi(s)}{\xi(0)}= \frac{det(H+1/4+s(1-s)}{det(H+1/4)}$ with $ H=p^{2}+ V(x) $ and $ V^{-1} (x)= 2 \sqrt \pi \frac{d^{1/2}}{dx^{1/2}}\frac{1}{\pi}arg\xi(1/2+i\sqrt x)$

This post imported from StackExchange Physics at 2014-03-24 09:19 (UCT), posted by SE-user Jose Javier Garcia
answered Aug 22, 2012 by Jose Javier Garcia (70 points) [ no revision ]
+ 1 like - 0 dislike

There's the Langlands program in supersymmetric quantum gauge theories, and string theory.

This post imported from StackExchange Physics at 2014-03-24 09:19 (UCT), posted by SE-user Nupcare
answered Aug 25, 2012 by Nupcare (10 points) [ no revision ]

Your answer

Please use answers only to (at least partly) answer questions. To comment, discuss, or ask for clarification, leave a comment instead.
To mask links under text, please type your text, highlight it, and click the "link" button. You can then enter your link URL.
Please consult the FAQ for as to how to format your post.
This is the answer box; if you want to write a comment instead, please use the 'add comment' button.
Live preview (may slow down editor)   Preview
Your name to display (optional):
Privacy: Your email address will only be used for sending these notifications.
Anti-spam verification:
If you are a human please identify the position of the character covered by the symbol $\varnothing$ in the following word:
p$\hbar$ysicsOv$\varnothing$rflow
Then drag the red bullet below over the corresponding character of our banner. When you drop it there, the bullet changes to green (on slow internet connections after a few seconds).
Please complete the anti-spam verification




user contributions licensed under cc by-sa 3.0 with attribution required

Your rights
...